The Student Room Group

integration question +r

Hi, I've done most of this question, could someone tell me if I'm doing it right please. Rep

Question: Evaluate the line integral of:
2ydyyx3dx \oint 2ydy-yx^3dx with C as the triangle with vertices (0,0), (0,1) and (1,1)

Ive integrated from (0,0) to (1,1) by taking y=x, 0<x<1 to get 7/15 [not sure if right]

Ive integrated from (1,1) to (0,1) by taking y=1, 0<x<1 to get -1/4 [not sure if right]

And for the last part integrating from (0,1) to (0,0) I had x=0 so dx=0, and 0<y<1 but I'm not sure if I can now integrate wrt y, if so I get 1.

So as the answer I'd get 7/15 - 1/4 +1 = 73/60 [1.216]


Thanks
Reply 1
Hmm...

Integrating from (0,0) to (1,1). We can use the parameter x=y=tx=y=t, t[0,1]t \in [0,1]

This gives the integral, 012t1dt01t41dt=01(2tt4)dt=45\displaystyle \int_0^1 2t \cdot 1 \, dt - \int_0^1 t^4 \cdot 1 \, dt = \int_0^1 (2t-t^4) \, dt = \boxed{\frac{4}{5}}

I'm not sure exactly what you've done?
Reply 2
Ah yeah for that bit I did the same method but had a sign error, now I get 4/5.

How about the rest?
Reply 3
I disagree with your sign for the second one.

We parametrise the path from (1,1) to (0,1) as x=1-t, y=1, t[0,1]t \in [0,1]

Therefore we get the integral

0121(0)dt011(1t)3(1)dt=01(1t)3dt=14\displaystyle \int_0^1 2 \cdot 1 \cdot ( 0 ) \, dt - \int_0^1 1 \cdot (1-t)^3 \cdot (-1) \, dt = \int_0^1 (1-t)^3 \, dt = \frac{1}{4}?

I also disagree with your sign on the last part
Reply 4
I am of course, assuming the direction is (0,0)->(1,1)->(0,1)
Reply 5
SimonM
I disagree with your sign for the second one.

We parametrise the path from (1,1) to (0,1) as x=1-t, y=1, t[0,1]t \in [0,1]

Therefore we get the integral

0121(0)dt011(1t)3(1)dt=01(1t)3dt=14\displaystyle \int_0^1 2 \cdot 1 \cdot ( 0 ) \, dt - \int_0^1 1 \cdot (1-t)^3 \cdot (-1) \, dt = \int_0^1 (1-t)^3 \, dt = \frac{1}{4}?

I also disagree with your sign on the last part



Hmm, the way I did it was [for the second part [and yes the direction is correct]]:
y=1, 0<x<1 so dy=0 [no change in y] so 2ydyyx3dx \int 2ydy - yx^3dx goes to 012dyx3dx=01x3dx=14 \int_0^1 2dy - x^3 dx = \int_0^1-x^3 dx =-\frac{1}{4}

I'm also following methods my lecturer used just to let you know =) Thanks for the help so far btw.
Reply 6
Shadow!

I'm also following methods my lecturer used just to let you know =) Thanks for the help so far btw.


Remember, the "direction" of x is ranging from 1 to 0, rather than 0 to 1
Reply 7
SimonM
Remember, the "direction" of x is ranging from 1 to 0, rather than 0 to 1


So if the direction goes from 1 to 0 instead of 0 to 1, then we integrate with the integrating values switched? So 01F(x)dx=10F(x)dx \int_0^1 F(x) dx = -\int_1^0 F(x) dx which would be why we had opposite signs...
Reply 8
Yes, exactly.

That said, your professor might prefer you to say something about dx. I don't know how he's taught you
Reply 9
SimonM
Yes, exactly.

That said, your professor might prefer you to say something about dx. I don't know how he's taught you


I think you may be right about that. That being said, is what I have done for the third and last part of the curve correct? Is it right to integrate with respect to y after doing the first two parts with respect to x?
Reply 10
Shadow!
I think you may be right about that. That being said, is what I have done for the third and last part of the curve correct? Is it right to integrate with respect to y after doing the first two parts with respect to x?


Well, as you said in the second part "dy = 0", we do exactly the same here with x, "dx = 0". Then proceed with integration with y. (Be careful about the direction mind)

Latest